Use non-breaking space (~) between 'Figure' and the figure number.
[course.git] / latex / problems / Serway_and_Jewett_8 / problem11.05.tex
1 \begin{problem*}{11.5}
2 Caclulate the net torque (magnitude and direction) on the beam in
3 Figure~P11.5 about \Part{a} an axis through $O$ perpendicular to the
4 page and \Part{b} an axis through $C$ perpendicular to the page.
5 \end{problem*}
6
7 \begin{solution}
8 \Part{a}
9 \begin{equation}
10   \sum \tau =   0\U{m}\cdot30\U{N}
11               + 2.0\U{m}\cdot25\U{N}\sin(90\dg-30\dg)
12               + 4.0\U{m}\cdot10\U{N}\sin(-20\dg)
13     = \ans{29.6\U{J}} \;.
14 \end{equation}
15 Because we used the usual right handed cross product, the positive
16 torque will cause counter clockwise rotation.
17
18 \Part{b}
19 \begin{equation}
20   \sum \tau =   2\U{m}\cdot30\U{N}\sin(45\dg)
21               + 0\U{m}\cdot25\U{N}
22               + 2.0\U{m}\cdot10\U{N}\sin(-20\dg)
23     = \ans{35.6\U{J}} \;.
24 \end{equation}
25 As in \Part{a}, positive torques are counter clockwise.
26 \end{solution}